Download as pdf or txt
Download as pdf or txt
You are on page 1of 62

CHASE ACADEMY

Near Vaivakawn Presbyterian Church


Aizawl, Mizoram
Contact: 8413829638/8794804104

PRELIMS MOCK TEST SERIES, 2024


POLITY
Time Allowed : 2 hours Maximum Marks : 200

INSTRUCTIONS
1. IMMEDIATELY AFTER THE COMMENCEMENT OF THE EXAMINATION, YOU SHOULD CHECK THAT THIS TEST
BOOKLET DOES NOT HAVE ANY UNPRINTED OR TORN OR MISSING PAGES OR ITEMS, ETC. IF SO, GET IT
REPLACED BY A COMPLETE TEST BOOKLET.

2. This Test Booklet contains 100 questions. Each item comprises four responses (answers). You will select
the response which you want to mark on the Answer Sheet. In case you feel that there is more than one
correct response, mark the response which you consider the best. In any case, choose ONLY ONE response
for each item.

3. You have to mark all your responses ONLY on the OMR Sheet provided to you.

4. All questions carry equal marks.THERE WILL BE PENALTY FOR WRONG ANSWERS MARKED BY A CANDIDATE
IN ALL THE QUESTIONS.

i. There are four alternatives for the answer to every question. For each question for which a wrong
answer is given by the candidate, one-third of the marks assigned to that question will be deducted
as penalty.

ii. If a candidate gives more than one answer, it will be treated as a wrong answer even if one of the
given answers happens to be correct and there will be same penalty as above to that question.

iii. If a question is left blank, i.e., no answer is given by the candidate, there will be no penalty for that
question.
1. Which of the following is a feature of the Government of India Act 1935?
A. It introduced dyarchy in the provinces and abolished dyarchy at the Centre
B. It abolished dyarchy in the provinces and introduced dyarchy at the Centre
C. It divided the powers of the Centre and units in terms of Federal List and Provincial List
D. It introduced bicameralism in all eleven provinces
Explanation: The GoI Act, 1935 divided the powers of the centre and units in terms of three lists – Federal
list, Provincial List and Concurrent List. It introduced dyarchy at the Centre and abolished it in the
provinces. Bicameralism was introduced in 6 out of 11 provinces.

2. Which of the following is correct about the Interim Government (1946)?


A. The Viceroy continued to be head of the Council
B. The members of the Interim Government were elected
C. Jawaharlal Nehru was designated as President
D. All of the statements are correct
Explanation: The members of the Interim Government were members of the Viceroy’s Executive
Council. The Viceroy continued to head of the Council while Jawaharlal Nehru was designated as Vice
President of the Council.

3. Which of the following pairs are correct in the context of the First Cabinet of Free India?
Member Portfolio

I. Dr. B R Ambedkar Labour


II. Dr. Rajendra Prasad Food & Agriculture
III. Maulana Abul Kalam Azad Education
IV. Sardar Vallabhbhai Patel Defence
A. I and II
B. II and III
C. III and IV
D. I, II, III & IV
Explanation: Dr. B R Ambedkar – Law; Sardar Vallabhbhai Patel – Home, Information & Broadcasting,
States

4. Consider the following statements about the Constituent Assembly


I. The Assembly had two Vice Presidents
II. The first meeting was attended by 211 members
III. The first meeting was held on December 11, 1946
IV. The Objectives Resolution was moved on December 14, 1946
Which of the given statements are correct?

A. I and II
B. II and III
C. III and IV
D. I, II, III and IV
Explanation: The Constituent Assembly held its first meeting on December 9, 1946 which was attended
by only 211 members as it was boycotted by the Muslim League. Both HC Mukherjee and VT
Krishnamachari were elected as Vice Presidents. The Objectives Resolution was moved by Jawaharlal
Nehru on December 13, 1946.

5. Which of the following was not a member of the Cabinet Mission?


A. Lord Penthick Lawrence
B. Sir Stafford Cripps
C. AV Alexander
D. Lord Wavell
Explanation: The Cabinet Mission Plan was a statement made by the Cabinet Mission and the Viceroy,
Lord Wavell, on May 16, 1946, that contained proposals regarding the constitutional future of India in
the wake of Indian political parties and representatives not coming to an agreement. The members of
the Cabinet Mission were: Lord Penthick-Lawrence, Secretary of State for India, Sir Stafford Cripps,
President of the Board of Trade, and A.V Alexander, First Lord of Admiralty.

6. Which of the following pairs do not match?

Sources Feature borrowed

I. Japan Procedure established by law


II. Australia Suspension of Fundamental Rights during emergency
III. South Africa Procedure for amendment
IV. Canada Concurrent List
A. I, II, III
B. II, III, IV
C. I and III
D. II and IV
Explanation: The suspension of fundamental rights during emergency is borrowed from the Weimer
Constitution of Germany. The Concurrent List is a feature borrowed from the Australian Constitution.

7. Choose the option that best describes a referendum


A. A method by which the people can propose a bill to the legislature for enactment
B. A method of obtaining public opinion on an issue of public importance
C. A procedure whereby a proposed legislation is referred to the electorate for settlement by
direct voting
D. A method by means of which voters can remove a representative before expiry of his/her term
Explanation: Option A describes an Initiative, Option B describes a Plebiscite and Option D describes a
Recall. These are all instruments of direct democracy.

8. The instrument of instructions contained in the Government of India Act 1935 have been incorporated
in the Constitution as
A. Conduct of Business Rules
B. Directive Principles of State Policy
C. Rules of Procedure for both Houses of Parliament
D. Fundamental Duties
Explanation: The `Instrument of Instructions` was a set of guidelines issued to the Governors of
Provinces in British India for the exercise of their discretionary powers. These guidelines were
incorporated into the Constitution of India in 1950 as `Directive Principles of State Policy`. The Directive
Principles are non-justiciable in nature, meaning they cannot be enforced by the courts. They are aimed
at guiding the State in making laws and policies that promote the welfare of the people and establish a
just and equitable society.

9. Consider the following statements with respect to Freedom of Religion enshrined in the Constitution:
1 It includes the freedom to not follow any religion.
2 It bars all religious conversions.
3 It is not applicable to foreign nationals.
Which of the statements given above is/are correct?

A. 1 only
B. 2 only
C. 1 and 3 only
D. 2 and 3 only
Explanation: Freedom of Religion is a Fundamental Right provided under Articles 25-28 of the
Constitution. It includes the freedom of conscience which means that a person may choose any religion
or may choose not to follow any religion. Hence, statement 1 is correct.

Freedom of Religion includes the right to not just practice one’s religion but also to propagate it. This
includes persuading people to join one’s religion and wilful conversion from one religion to another. The
Constitution bars forceful conversions and conversions done by inducements. Hence, statement 2 is not
correct.

Freedom of Religion is available to not just Indian citizens but also to foreign nationals residing within
Indian territory. Hence, statement 3 is not correct.

10. Which of the following are the similarities between Indian Parliamentary System and British
Parliamentary System?
1. Supremacy of Parliament

2. Elected head of state

3. Majority party rule

Select the correct answer using the code given below.

A. 1 and 3 only
B. 1 and 2 only
C. 3 only
D. None
Explanation: The features of parliamentary government in India are:

• Presence of nominal and real executives;


• Majority party rule,
• Collective responsibility of the executive to the legislature,
• Membership of the ministers in the legislature,
• Leadership of the prime minister or the chief minister, Dissolution of the lower House (Lok Sabha
or Assembly).
There are some differences between the British Parliamentary system and that of India. For example,
the Indian Parliament is not a sovereign body like the British Parliament. Also, the Indian State has an
elected head (republic) while the British State has hereditary head (monarchy).

11. Consider the following functionaries of NITI Aayog:


1. Chairperson

2. Vice-chairperson

3. Special Invitees

4. Chief Executive Officer

Which of the above are appointed by the Prime Minister?

A. 2, 3 and 4 only
B. 1, 2 and 3 only
C. 2 and 4 only
D. 1, 3 and 4 only
Explanation: The Composition of the NITI Aayog is as follows:

• Chairperson: The Prime Minister of India


• Governing Council: It comprises the Chief Ministers of all the States, Chief Ministers of Union
Territories with Legislatures (i.e., Delhi and Puducherry) and Lt. Governors of other Union
Territories.
• Regional Councils: These are formed to address specific issues and contingencies impacting more
than one state or a region. These are formed for a specified tenure. These are convened by the
Prime Minister and comprises of the Chief Ministers of States and Lt. Governors of Union
Territories in the region. These are chaired by the Chairperson of the NITI Aayog or his nominee.
• Special Invitees: Experts, specialists and practitioners with relevant domain knowledge as special
invitees nominated by the Prime Minister.
• Full-time Organisational Framework: It comprises, in addition to the Prime Minister as the
Chairperson, the Vice-Chairperson, Full-time members, ex-officio members, CEO with a
Secretariat.

12. Under Article 360 of the Constitution, the power to proclaim financial emergency is vested under
A. Ministry of Finance
B. Parliament
C. President
D. Comptroller and Auditor General of India
Explanation: Article 360 empowers the President to proclaim a Financial Emergency if he is satisfied that
a situation has arisen due to which the financial stability or credit of India or any part of its territory is
threatened. The 38th Amendment Act of 1975 made the satisfaction of the President in declaring a
Financial Emergency final and conclusive and not questionable in any court on any ground. But, this
provision was subsequently deleted by the 44th Amendment Act of 1978 implying that the satisfaction
of the President is not beyond judicial review.

13. Indian form of government is described as a federation with centralizing tendency. Which among the
following provisions of the constitution supports this tendency?
1. A single and flexible constitution

2. Provision of All India Services

3. Appointment of State Governor by the Centre.

Select the correct answer using the code given below.

A. 1 and 2 only
B. 2 and 3 only
C. 1 and 3 only
D. 1, 2 and 3
Explanation: Unlike the USA, where it has two sets of constitutions for center and states, India has a
single constitution for both states and center. The Indian constitution is also flexible unlike the case of
a true federation where it will be rigid. This is a centralizing tendency as states boundaries can be altered
by the center.

The candidates selected for All India services were recruited and trained by the center but are to be
served in the states, which do not have the capacity to 'remove' them.

The governor having many discretionary powers at the state is appointed by the center. This is one of
the major centralizing tendencies.
14. Which of the following may be drawbacks of a democracy?
1. Leaders keep changing in a democracy which could lead to instability.

2. Democracy may lead to corruption as it is based on electoral competition.

3. Democracy increases the chances of rash or irresponsible decision-making.

Select the correct answer using the code given below.

A. 1 and 2 only
B. 3 only
C. 1 only
D. 1, 2 and 3

Explanation: Democracy as a form of government only ensures that people take their own decisions.
This does not guarantee that their decisions will be good. People can make mistakes. Involving the
people in these decisions does lead to delays in decision making. It is also true that democracy leads to
frequent changes in leadership. Sometimes this can set back big decisions and affect the government’s
efficiency.

Democracy may lead to corruption for it is based on electoral competition due to more expenditure in
elections, criminalisation of politics, etc.

Democracy is based on consultation and discussion. A democratic decision always involves many
persons, discussions and meetings. When a number of people put their heads together, they are able
to point out possible mistakes in any decision. This takes time. But there is a big advantage in taking
time over important decisions. This reduces the chances of rash or irresponsible decisions. Thus,
democracy improves the quality of decision-making.

15. Consider the following statements regarding the National Population Register (NPR):

1. Only Citizens are included in the NPR.

2. It is mandatory for every usual resident of India to register in the NPR.

3. The data for NPR was first collected in 2010 along with the house-listing phase of Census 2011.

Which of the statements given above is/are correct?

A. 1 only
B. 2 and 3 only
C. 3 only
D. 1, 2 and 3
Explanation: The NPR is a list of “usual residents of the country”. According to the Home Ministry, a
“usual resident of the country” is one who has been residing in a local area for at least the last six months
or intends to stay in a particular location for the next six months.

NPR is not a citizenship enumeration drive, as it would record even a foreign national staying in a locality
for more than six months. This makes NPR different from the NRC, which includes only Indian citizens
while seeking to identify and exclude non-citizens. Hence statement 1 is not correct.

The NPR is being prepared under provisions of the Citizenship Act, 1955 and the Citizenship (Registration
of Citizens and issue of National Identity Cards) Rules, 2003.

It is mandatory for every “usual resident of India” to register in the NPR. Only Assam will not be included
(as per a notification by the Registrar General of India in August), given the recently completed NRC in
that state

NPR will be conducted in conjunction with the house-listing phase, the first phase of the Census, by the
Office of Registrar General of India (RGI) for Census 2021.

It is conducted at the local, sub-district, district, state and national levels. The data for NPR was first
collected in 2010 along with the house-listing phase of Census 2011. In 2015, this data was updated by
conducting door-to-door surveys.

16. Consider the following statements:


1. There is no limit to the power of the Parliament to amend the Constitution.

2. Only the President can decide whether any matter forms a part of the basic structure of the
Constitution

Which of the statements given above is/are correct?

A. 1 only
B. 2 only
C. Both 1 and 2
D. Neither 1 nor 2
Explanation: In 1973, the Supreme Court ruled in Kesavananda Bharati case that there is a basic
structure of the Constitution and nobody—not even the Parliament (through amendment)—can violate
the basic structure. The Court did two more things. First, it said that right to property (the disputed
issue) was not part of basic structure and therefore could be suitably abridged. Secondly, the Court
reserved to itself the right to decide whether various matters are part of the basic structure of the
Constitution. This case is perhaps the best example of how judiciary uses its power to interpret the
Constitution
17. Which of the following statements is/are correct regarding Fundamental Rights?
1. They are sacrosanct in nature and cannot be amended.

2. They are available against the actions of both State and private individuals.

3. They operate as checks on the tyranny of both executive and the Legislature.

Select the correct answer using the code given below.

A. 1 and 3 only
B. 2 only
C. 2 and 3 only
D. 1, 2 and 3
Explanation: Fundamental rights can be amended by constitution amendment till they do not violate
the basic structure of the constitution and thus they are not sacrosanct.

They are available against the actions of both State and private individuals. Few Rights like Abolition
of Untouchability etc are available against private citizens also.

These rights limit the power of the Executive and legislature and thus prevent tyranny of the
executive and legislature.

18. Consider the following statements regarding the Asian Infrastructure Investment Bank:
1. India is the second-largest shareholder of the bank.

2. Both USA and Japan are not its members.

Which of the statements given above is/are correct?

A. 1 only
B. 2 only
C. Both 1 and 2
D. Neither 1 nor 2
Explanation: The Asian Infrastructure Investment Bank (AIIB) is a multilateral development bank that
aims to improve economic and social outcomes in Asia.

AIIB has 105 members and is open to accepting additional members. AIIB is headquartered in Beijing,
China. The AIIB, launched in Beijing in 2015, has approved more loans for India than any other
member of the bank. The AIIB has funded 28 projects in India amounting to $6.7 billion. Initially, 70-
80% of the projects were co-financed but now that share of projects is standalone.

China is its biggest shareholder and India is the second-largest. The U.S.A and Japan are not among
its members.
19. Which of the following statements is/are correct with regard to the Fundamental Duties?
1. They can be used by the courts to determine the constitutionality of a law.

2. They can be enforced by the Parliament through legislation.

Select the correct answer using the codes given below.

A. 1 only
B. 2 only
C. Both 1 and 2
D. Neither 1 nor 2
Explanation: Supreme Court in 1992 stated that Fundamental duties can be used in determining the
constitutionality of any law.

Parliament is free to enforce Fundamental duties through suitable legislation. Many legislations like
the Prevention of Insults to National Honour Act, 1971 make insult to national symbols a punishable
act.

20. Which of the following Supreme Court cases is/are related to disputes/conflicts between Fundamental
Rights and Directive Principles of State Policy?
1. IR Coehlo Case, 2007

2. Golaknath case, 1967

3. Maneka Gandhi case, 1978

4. Minerva Mill Case, 1980

Select the correct answer using the code given below.

A. 1 and 2 only
B. 2 and 4 only
C. 3 and 4 only
D. 1 only
Explanation: Cases related to dispute/conflict between Fundamental Right and Directive Principles
of State Policy (DPSPs):

• Champakam Dorairajan Case, 1951 - In this case, Supreme Court ruled that in case of any conflict
between FRs and DPSPs, FRs would prevail. It declared that DPSPs have to conform to and run as
subsidiaries to the fundamental right. However, it also held that FRs could be amended by the
Parliament through constitutional amendment acts. This led to the First, Fourth, and
Seventeenth Amendment Act to implement some of the DPSPs.
• Golak Nath case, 1967 - In this case, Supreme Court held that Parliament can’t take away or
abridge any of the Fundamental Rights, which are ‘sacrosanct’ in nature. Hence, the court held
that Fundamental Rights can’t be amended for the implementation of DPSPs. This led to the
enactment of the 24th Amendment Act and the 25 amendment Act which inserted a new Article
31C.
• Kesavanand Bharti case, 1973 - In this case, Supreme Court declared the second provision of
Article 31C as unconstitutional and invalid on the ground that judicial review is a basic feature of
the Constitution. This led to the enactment of the 42nd Amendment Act which gave legal primacy
and supremacy to the DPSPs over FRs conferred by Articles 14, 19, and 21.
• Minerva Mills case, 1980 - In this case, Supreme Court held the primacy of DPSPs over FRs as
unconstitutional and invalid. It led to the subordination of DPSPs over FRs. However, FRs
conferred by Articles 14 and 19 were accepted as subordinate to the DPSPs specified under
Article 39(b) and (c).
The present position is that the FRs enjoy supremacy over DPSPs. However, Parliament can amend
the FRs for implementing the DPSP, so long as the amendment doesn’t destroy of the basic feature
of the constitution.

• Maneka Gandhi case, 1978 - It deals with rights guaranteed under Article 21 of the
Constitution. Prior to this, there was ‘procedure prescribed by law’ which was replaced by
‘due process of law’ with respect to article 21 of the Constitution.
• IR Coelho case in 2007—popularly known as the Ninth Schedule case—the Supreme Court
took this further and argued that if the purpose of inserting a law into the Ninth Schedule was
to undo a judgment of the Supreme Court, this could be examined by the courts. Also, the
Supreme Court held that the laws placed under IX schedule after Kesavananda Bharati's
judgment (24th April 1973) cannot be exempt from Judicial review.

21. Which of the following would fall under the definition of State as defined in Article 12 of the Constitution?
1. Delhi Metro Rail Corporation

2. Unique Identification Authority of India

3. NITI Aayog

4. RBI

Select the correct answer using the code given below.

A. 1 and 3 only
B. 2 and 4 only
C. 1, 2, and 3 only
D. 1, 2, 3 and 4
Explanation: The state has been defined in a wider sense so as to include all its agencies. It is the
actions of these agencies that can be challenged in the courts as violating the Fundamental Rights.
According to the Supreme Court, even a private body or an agency working as an instrument of the
State falls within the meaning of the 'State' under Article 12.

• DMRC has equal equity participation from GOI and GNCTD.


• RBI, UID Authority are statutory bodies.
• NITI Aayog performs important public functions like recommending the poverty line.

22. Which of the following are the reasons for creating Union Territories in India?
1. Cultural distinctiveness

2. Strategic importance

3. The interest of tribal people

4. Administrative consideration

Select the correct answer using the code given below.

A. 1, 2 and 3 only
B. 1 and 4 only
C. 2, 3 and 4 only
D. 1, 2, 3 and 4
Explanation: The Union Territories in India have been created for a variety of reasons. These are
mentioned below:

• Political and administrative consideration-Delhi and Chandigarh.


• Cultural distinctiveness-Puducherry, Dadra and Nagar Haveli, and Daman and Diu.
• Strategic importance-Andaman and the Nicobar Islands and Lakshadweep.
• Special treatment and care of the backward and tribal people-Mizoram, Manipur, Tripura,
and Arunachal Pradesh which later became states.

23. Which of the following issues can be taken up by the National Human Rights Commission (NHRC)?
1. Issues of manual scavenging

2. Problems faced by Notified and Denotified Tribes

3. Issues related to the right to health

Select the correct answer using the codes given below.

A. 1 only
B. 1 and 2 only
C. 1, 2 and 3
D. 3 only
Explanation: The National Human Rights Commission (NHRC) of India is a Statutory public body
constituted under the Protection of Human Rights Act, 1993. The various human rights issues taken
up by the Commission are as follows:

• Abolition of Bonded Labour


• Issues Concerning Right to Food
• Protocols to the Convention on the Rights of the Child
• Abolition of Child Labour
• Trafficking in Women and Children
• Maternal Anaemia and Human Rights
• Combating Sexual Harassment of Women at the Work Place
• Abolition of Manual Scavenging
• Dalits issues including Atrocities perpetrated on them
• Problems faced by Denotified and Nomadic Tribes.
• Rights of the Disabled Persons
• Issues related to Right to Health.
• Rights of persons affected by HIV/AIDS

24. Which of the following Directive Principles was added by the 42nd Constitution amendment?
1. To secure opportunities for the healthy development of children.

2. To promote equal justice and to provide free legal aid to the poor.

3. To minimize inequalities in income, status, facilities, and opportunities.

4. To promote the development of the Hindi language.

Select the correct answer using the code given below.

A. 1 and 2 only
B. 2 and 3 only
C. 1, 2, and 4 only
D. 1, 2, 3 and 4
Explanation:

42nd, 1976

• To secure opportunities for healthy development of children (Article 39)


• To promote equal justice and to provide free legal aid to the poor (Article 39 A)
• To take steps to secure the participation of workers in the management of industries (Article
43 A)
• To secure opportunities for healthy development of children (Article 39)
• To protect and improve the environment and to safeguard forests and wild life (Article 48 A).
44th, 1978

• The State shall, in particular, strive to minimize the inequalities in income, and endeavour to
eliminate inequalities in status, facilities and opportunities, not only amongst individuals but
also amongst groups of people residing in different areas or engaged in different vocations
(Article 38(2)
97th, 2011

• The State shall endeavour to promote voluntary formation, autonomous functioning,


democratic control and professional management of co-operative societies. (Article 43-B)
86th, 2002

• The State shall endeavour to provide early childhood care and education for all children until
they complete the age of six years. (The subject of article 45 changed)

25. Who among the following do not hold office during the pleasure of the President?
1. Attorney General

2. Comptroller and Auditor General

3. Members of Union Public Service Commission

4. Chief Election Commissioner

Select the correct answer using the code given below.

A. 1 and 2 only
B. 2, 3 and 4 only
C. 1, 3 and 4 only
D. 1, 2 and 4 only
Explanation: The doctrine of Pleasure of President has been borrowed from the British. In England,
The doctrine of pleasure means that the Crown has the power to terminate the services of a civil
servant at any time they want without giving any notice of termination to the servant. Thus the civil
servants work at the pleasure of the Crown which can remove them at any time. This doctrine is
based on the concept of public policy and whenever the Crown feels that a civil servant should be
removed from his office because keeping him will be against public policy, the Crown can remove
such servant.
India has adopted this provision though there exist some modifications in case of the civil servants.

Article 310

• The term of office of the AG is not fixed by the Constitution. Further, the Constitution does not
contain the procedure and grounds for his removal. He holds office during the pleasure of the
president.

• CAG holds office for a period of six years or up to the age of 65 years, whichever is earlier. He can
resign any time from his office by addressing the resignation letter to the president. He can also be
removed by the president on same grounds and in the same manner as a judge of the Supreme Court.

In other words, he can be removed by the president on the basis of a resolution passed to that effect
by both the Houses of Parliament with a special majority, either on the ground of proved
misbehaviour or incapacity. Thus, he does not hold his office until the pleasure of the president,
though he is appointed by him.

• The President can remove the chairman or any other member of UPSC from the office under the
following circumstances:

o If he is adjudged an insolvent (that is, has gone bankrupt)

o If he engages, during his term of office, in any paid employment outside the duties of his office; or

o If he is, in the opinion of the president, unfit to continue in office by reason of infirmity of mind or
body.

o In addition to these, the President can also remove the chairman or any other member of UPSC for
misbehaviour. However, in this case, the president has to refer the matter to the Supreme Court for
an inquiry. If the Supreme Court, after the inquiry, upholds the cause of removal and advises so, the

President can remove the chairman or a member. Under the provisions of the Constitution, the
advice tendered by the Supreme Court in this regard is binding on the President. Thus,
Members/Chairman do not hold his office until the pleasure of the president, though they are
appointed by him.

The chief election commissioner is provided with the security of tenure. He cannot be removed from
his office except in the same manner and on the same grounds as a judge of the Supreme Court. In
other words, he can be removed by the President on the basis of a resolution passed to that effect
by both the Houses of Parliament with a special majority, either on the ground of proved
misbehaviour or incapacity. Thus, he does not hold his office until the pleasure of the President,
though he is appointed by him.

26. With reference to State Election Commission, consider the following statements:
1. It has been empowered to conduct elections to panchayats through 73rd Constitutional amendment.

2. The functions of delimitation, reservation and rotation of Panchayats seats are vested in State
Election Commission.

3. State Election Commission submits its annual report to the Election Commission of India and to the
Governor.

Which of the statements given above is/are correct?

A. 1 and 2 only
B. 2 only
C. 1 and 3 only
D. 1, 2 and 3
Explanation: State Election Commissions is entrusted to conduct elections to Panchayats through
73rd and urban local bodies through 74th Constitutional amendment acts.

The functions of delimitation, reservation and rotation of Panchayats seats is vested in State Election
Commission. As per the recommendations of National Commission to Review the Working of the
Constitution (NCRWC), the functions of delimitation, reservation and rotation of seats should be
vested in a Delimitation Commission and not in the State Election Commission.

At present, there is no clear-cut provision regarding the submission of reports by State Election
Commission. NCRWC also recommended that the State Election Commission should submit its
annual or special reports to the Election Commission of India and to the Governor.

27. Which of the following statements is/are correct regarding Article 17 (Abolition of Untouchability)?
1. It abolishes untouchability and its practice in any form.

2. "Untouchability" has been defined under Article 17.

3. It is available only against the state.

4. It is implemented by the "Protection of Civil Rights Act, 1955".

Select the correct answer using the code given below.

A. 1 and 2 only
B. 1 and 4 only
C. 2, 3 and 4 only
D. 4 only
Explanation: Article 17 states that "Untouchability" is abolished and its practice in any form is
forbidden. Untouchability has not been defined by the constitution but various court judgments have
expanded its meaning. It is available against both the state and private individuals. Protection of Civil
Rights Act,1955 contains many provisions for this.

Article 17 only states that the enforcement of any disability arising out of "Untouchability" shall be
an offense punishable in accordance with the law. Protection of Civil Rights Act, 1955 contains the
detailed provisions for this.

28. Consider the following statements regarding 'State Public Service Commission':
1. One-half of the members of the commission should be persons from SC/ST and OBCs.

2. Members of the SPSC can be removed by the Governor of the respective state.

Which of the statements given above is/are correct?

A. 1 only
B. 2 only
C. Both 1 and 2
D. Neither 1 nor 2
Explanation: A State Public Service Commission consists of a chairman and other members appointed
by the governor of the state. The Constitution does not specify the strength of the Commission but
has left the matter to the discretion of the Governor. Further, no qualifications are prescribed for the
commission's membership except that one-half of the members of the commission should be such
persons who have held office for at least ten years either under the government of India or under
the Government of a state. The Constitution also authorizes the governor to determine the
conditions of service of the chairman and members of the Commission.

Although the chairman and members of an SPSC are appointed by the governor, they can be removed
only by the President (and not by the governor). The President can remove them on the same
grounds and in the same manner as he can remove a chairman or a member of the UPSC.

29. Which of the following commissions/committee accepted "language" as the criteria for the
reorganization of states?
1. Dhar Commission

2. JVP Committee

3. Fazl Ali Commission

Select the correct answer using the code given below.


A. 1 only
B. 1 and 2 only
C. 3 only
D. 2 and 3 only
Explanation: Both the Dhar commission and JVP committee rejected language as the basis for the
reorganization of states. Fazl Ali's commission broadly accepted language as the criteria for the
reorganization of states, however, it rejected one- language - state policy.

30. Which of the following elements form a part of ‘Basic Structure of the Constitution’ as laid down by
various court judgements?
1. Secular character of the Constitution

2. Separation of powers between the legislature and the executive

3. Socialist state

4. Principle of reasonableness

Select the correct answer using the code given below.

A. 1, 2 and 3 only
B. 2, 3 and 4 only
C. 1, 3 and 4 only
D. 1, 2 and 4 only
Explanation: The doctrine of Basic structure of the Constitution was laid down by the Supreme Court
in the case of Kesavananda Bharati v/s State of Kerala (1973). Based on various judgements of the
Court, the following are some of the elements included in the Basic structure: Supremacy of the
Constitution; Secular character of the Constitution; Sovereign, democratic and republic nature of
polity; Separation of powers between the legislature, judiciary and executive; Unity and Integrity of
the nation; Welfare state; Rule of Law; Parliamentary system; Effective access to justice; Principle of
reasonableness etc

Socialist state is not included in the Basic structure.

31. A religious denomination must satisfy which of the following necessary conditions in order to exercise
the freedom to manage religious affairs under Article 26?
1. Group of individuals with a system of beliefs.

2. A group with a common organization.

3. Collective has a distinctive name.


4. The group has a religious text of its own.

5. A leader who preaches a distinct doctrine.

Select the correct answer using the code given below.

A. 1, 2, and 3 only
B. 3, 4, and 5 only
C. 1, 2, and 5 only
D. 1, 2, 3, 4 and 5
Explanation: The Supreme Court has held that a religious denomination must satisfy three
conditions:

• It should be a collection of individuals who have a system of beliefs (doctrines), which they
regard as conducive to their spiritual well-being.
• It should have a common organization
• It should be designated by a distinctive name.

32. Consider the following statements with reference to martial law in India:
1. The Indian constitution under Article 34 defines martial law.

2. Both Martial Law and National Emergency can be imposed in any part of the country.

Which of the above-mentioned statements is/are correct?

A. 1 only
B. 2 only
C. Both 1 and 2
D. Neither 1 nor 2
Explanation: Martial law has not been defined anywhere in the Constitution. It is implicit. Also, there
is no specific or express provision in the Constitution that authorizes the executive to declare Martial
Law. However, Article 34 provides for the restrictions on fundamental rights while Martial Law is in
force in any area.

Both Martial Law and National Emergency can be imposed in some specific area. However, the scope
of National emergency is much more. It can be imposed in the whole country as well.

33. Consider the following statements in the context of Preamble to the Constitution:
1. The Preamble states that the Constitution derives its authority from the constituent assembly of India.

2. Indian constitution was the first constitution in the world, to begin with a Preamble.
3. Preamble is a part of the Constitution and gives clarity in interpretation of the constitution.

Which of the statements given above is/are correct?

A. 1 and 2 only
B. 3 only
C. 2 and 3 only
D. 1, 2 and 3
Explanation: The Constitution derives its authority from the people of India not from the constituent
assembly, this is mentioned in the preamble part of the constitution.

The American constitution was the first to begin with a Preamble. Many countries, including India,
followed the practice.

In the Keshavananda Bharti case, SC has held that preamble is a part of the Constitution. And if there
is any lack of clarity in the constitution, the judiciary turns to the Preamble in its interpretation of the
relevant provisions.

34. If the President of India exercises his power as provided under Article 359 of the Constitution, then
A. the President can promulgate ordinance for any state.
B. the powers of High Courts are subsumed by the Supreme Court.
C. the President is empowered to suspend the enforcement of some Fundamental Rights.
D. he may extend National Emergency only to few parts of the country and not the entire country.
Explanation: Article 359 authorises the President to suspend the right to move any court for the
enforcement of Fundamental Rights during a National Emergency. This means that under Article 359,
the Fundamental Rights as such are not suspended, but only their enforcement. The said rights are
theoretically alive but the right to seek remedy is suspended. The suspension of enforcement relates
to only those Fundamental Rights that are specified in the Presidential Order. Further, the suspension
could be for the period during the operation of emergency or for a shorter period as mentioned in
the order, and the suspension order may extend to the whole or any part of the country. It should
be laid before each House of Parliament for approval. Hence, only option (c) is correct.

The 44th Amendment Act of 1978 restricted the scope of Article 359 in two ways. Firstly, the
President cannot suspend the right to move the Court for the enforcement of fundamental rights
guaranteed by Articles 20 to 21. Secondly, only those laws which are related with the emergency are
protected from being challenged and not other laws and the executive action taken only under such
a law, is protected.

Article 359 operates in case of both when National Emergency is declared on grounds of war or
external aggression as well as on the ground of armed rebellion. Article 359 may extend to the entire
country or a part of it.
35. The Indian Councils Act of 1861 introduced which of the following provisions for the first time in India?
1. Power to issue ordinances in Indian administration

2. Nomination of Indians as non-official members to Viceroy’s council

3. The new office of Secretary of State for India

Select the correct answer using the code given below.

A. 1 only
B. 1 and 2 only
C. 2 and 3 only
D. 1, 2 and 3

Explanation: Indian Councils Act of 1861 empowered Viceroys to issue ordinances without the
concurrence of the legislative council, during an emergency. The life of such an ordinance was 6
months.

It made a beginning of representative institutions by associating Indians with the law-making


process. It thus provided that the viceroy should nominate some Indians as non-official members of
his expanded council. In 1862, Lord Canning, the then viceroy, nominated three Indians to his
legislative council-the Raja of Benaras, the Maharaja of Patiala, and Sir Dinkar Rao.

The office of Secretary of State for India was created under the provisions of the Act of 1858 (Act for
Good Governance of India)

36. The expression Union of India under Article 1 of the Indian constitution include(s) which of the following?
1. States

2. Union Territories

3. Territories that India may acquire in the future

Select the correct answer using the code given below.

A. 1 only
B. 1 and 2 only
C. 2 and 3 only
D. 1, 2 and 3
Explanation: 'Territory of India’ is a wider expression than the ‘Union of India’ because the latter
includes only states while the former includes not only the states but also union territories and
territories that may be acquired by the Government of India at any future time.
37. Indian Constitution is called a living document because
1. Indian Constitution is open to changes as per the changing needs of society.

2. The constitution provides enough flexibility of interpretation for both the political class and the
Judiciary.

Select the correct answer using the code given below.

A. 1 only
B. 2 only
C. Both 1 and 2
D. Neither 1 nor 2

Explanation:

38. With reference to the Solicitor General of India, consider the following statements:
1. The office of the Solicitor General of India is mentioned in the Indian Constitution.

2. The role of the Solicitor General is to assist the Attorney General in the fulfilment of his official
responsibilities.

Which of the statements given above is/are correct?

A. 1 only
B. 2 only
C. Both 1 and 2
D. Neither 1 nor 2
Explanation: In addition to the Attorney General, there are other law officers of the Government of
India. They are the Solicitor General of India and additional Solicitor General of India. They assist the
Attorney General in the fulfilment of his official responsibilities. It should be noted here that only the
office of the Attorney General is created by the Constitution. In other words, Article 76 does not
mention about the Solicitor General and additional Solicitor General.

39. Consider the following statements regarding the North Atlantic Treaty Organisation (NATO):

1. It is committed to the principle that an attack against one of its members is considered as an attack
against all.

2. Its headquarters is in Brussels, Belgium.

Which of the statements given above is/are correct?


A. 1 only
B. 2 only
C. Both 1 and 2
D. Neither 1 nor 2
Explanation: The North Atlantic Treaty Organization also called the North Atlantic Alliance, is an
intergovernmental military alliance between 27 European countries, 2 North American countries,
and 1 Eurasian country. The organization implements the North Atlantic Treaty that was signed on 4
April 1949. Its headquarters is located in Brussels, Belgium.

NATO is committed to the principle that an attack against one or several of its members is considered
as an attack against all. Hence statement 1 is correct. This is the principle of collective defense, which
is enshrined in Article 5 of the Washington Treaty. So far, Article 5 has been invoked once - in
response to the 9/11 terrorist attacks in the United States in 2001.

40. Consider the following statements with regard to Parliament's power to reorganize the states:
1. Any bill contemplating the changes can be introduced in the parliament only with the prior
recommendation of the President.

2. Under article 3, the Parliament’s power to diminish the areas of a state includes the power to cede
Indian territory to a foreign state.

3. Any bill aiming to reorganize the states shall be passed in the parliament by a special majority.

Which of the statements given above is/are correct?

A. 1 only
B. 1 and 2 only
C. 2 and 3 only
D. None
Explanation: Any bill contemplating the changes with regard to the reorganization of states can be
introduced in the parliament only with the prior recommendation of the President. Under article 3
of the Constitution, the parliament's power to diminish the areas of a state does not include the
power to cede Indian territory to a foreign state. Indian territory can be ceded to a foreign state only
by amending the Constitution under Article 368.

Any bill aiming to reorganize the states shall be passed in the parliament by a simple majority.

41. Consider the following statements:


1. Adjournment motion can be introduced in Lok Sabha only.
2. No confidence motion can be moved against an individual minister or the entire council of minister.

3. Censure motion can be moved against the entire council of minister only.

Which of the statements given above is/are not correct?

A. 1 and 2 only
B. 2 and 3 only
C. 1 and 3
D. 1, 2 and 3
Explanation: Adjournment motion can be introduced in Lok Sabha only. No confidence motion can
be moved against the entire council of minister only. Censure motion can be moved against an
individual minister or the entire council of ministers.

42. Which of the following is/are correct with reference to the Cantonment Board?
1. It is established for municipal administration for civilian population in the cantonment area.

2. It works under the administrative control of the Union Ministry of Home Affairs.

3. It consists of partly elected and partly nominated members.

Select the correct answer using the code given below.

A. 1 only
B. 2 and 3 only
C. 1 and 3 only
D. 1, 2 and 3
Explanation: A cantonment board is established for municipal administration for the civilian
population in the cantonment area. It is set up under the provisions of the Cantonments Act of
2006—legislation enacted by the Central government.

It works under the administrative control of the Defence Ministry of the Central government. Thus,
a cantonment board is created as well as administered by the Central government. A cantonment
board consists of partly elected and partly nominated members. The elected members hold office
for a term of five years while the nominated members (i.e., ex-officio members) continue so long as
they hold the office in that station. The military officer commanding the station is the ex-officio
president of the board and presides over its meetings. The Vice-President of the board is elected by
the elected members from amongst themselves for a term of five years.

43. Who among the following is responsible for the conduct of elections in the Parliamentary or assembly
constituency?
A. Returning Officer
B. Chief Electoral Officer
C. District Election Officer
D. Electoral Registration Officer
Explanation: The Returning Officer of a Parliamentary or assembly constituency is responsible for the
conduct of elections in the Parliamentary or assembly constituency concerned. The Election
Commission of India nominates or designates an officer of the Government or a local authority as
the Returning Officer for each of the assembly and parliamentary constituencies in consultation with
the State Government / Union Territory Administration. In addition, the Election Commission of India
also appoints one or more Assistant Returning Officers for each of the assembly and Parliamentary
constituencies to assist the Returning Officer in the performance of his functions in connection with
the conduct of elections.

44. Which of the following statements is correct regarding the salary and allowances of supreme court judges
in India?
A. They are determined by the President.
B. It can be altered by the Parliament to the judge's disadvantage after his appointment.
C. It is charged on the Consolidated Fund of State.
D. None of the above statements are correct.
Explanation: The Parliament determines the salary, other allowances, leave of absence, pension, etc.
of the Supreme Court judges. The Parliament cannot alter any of these privileges and rights to the
judge's disadvantage after his appointment. Salaries, allowances, and pensions of the judges of the
Supreme Court are charged on the Consolidated Fund of India.

45. With reference to Vice President of India, which of the following statements is/are correct?
1. Vice President is elected by the same electoral college as that of the President.

2. A resolution for his removal need to be passed with special majority in both the houses.

Select the correct answer using the code given below.

A. 1 only
B. 2 only
C. Both 1 and 2
D. Neither 1 nor 2
Explanation: The method of election of the Vice President is similar to that of the President. The only
difference is that members of State legislatures are not part of the Electoral College.
The Vice President acts as the ex-officio Chairman of the Rajya Sabha and takes over the office of the
President when there is a vacancy by reasons of death, resignation, removal by impeachment or
otherwise.

Removal of the Vice-President of India: Article 67 of the Indian Constitution states that VicePresident
can be removed from office before the completion of his term. A formal impeachment is not required
for his removal. He can be removed by a resolution of the Rajya Sabha passed by a majority of all the
then members of the Rajya Sabha and agreed to by the Lok Sabha. This means that this resolution
should be passed in the Rajya Sabha by an effective majority and in the Lok Sabha by a simple
majority. But, no such resolution can be moved unless at least 14 days advance notice has been given.
No ground has been mentioned in the Constitution for his removal.

46. Consider the following statements regarding Committee on Public undertakings:


1. The chairman of this Committee can be only from Lok Sabha.

2. The members are elected every year for a term of one year.

Which of the statements given above is/are correct?

A. 1 only
B. 2 only
C. Both 1 and 2
D. Neither 1 nor 2
Explanation: The Committee on Public Undertakings committee was created in 1964 on the
recommendation of the Krishna Menon Committee. The functions of the committee are:

• To examine the reports and accounts of public undertakings.


• To examine the reports of the Comptroller and Auditor General on public undertakings.
• The committee is not to examine and investigate any of the following:
✓ Matters of major government policy as distinct from business or commercial functions of
the public undertakings;
✓ Matters of day-to-day administration;
✓ Matters for the consideration of which machinery is established by any special statute
under which a particular public undertaking is established;
✓ Technical matters as its members are not technical experts.
The chairman of the committee is appointed by the Speaker from amongst its members who are
drawn from the Lok Sabha only. The members are elected every year for a term of 1 year. The
elections are done through proportional representation using a single transferable vote. A minister
cannot be elected as a member of the committee.
47. With reference to the evolution of Panchayati Raj in India, consider the following statements:
1. The Balwant Rai Mehta Committee recommended establishment of a two- tier Panchayati Raj
system.

2. Rajasthan was the first state in independent India to establish Panchayati Raj.

3. The Panchayati Raj institutions got constitutional status in 1982.

Which of the statements given above is/are correct?

A. 1 and 2 only
B. 2 only
C. 1 and 3 only
D. 1, 2 and 3

Explanation: In January 1957, the Government of India appointed a committee to examine the
working of the Community Development Programme (1952) and the National Extension Service
(1953) and to suggest measures for their better working. The chairman of this committee was
Balwant Rai G Mehta. The committee submitted its report in November 1957 and recommended the
establishment of the scheme of ‘democratic decentralization, which ultimately came to be known as
Panchayati Raj. It recommended the establishment of a three-tier Panchayati raj system—gram
panchayat at the village level, panchayat samiti at the block level and zila parishad at the district
level.

Rajasthan was the first state to establish Panchayati Raj. The scheme was inaugurated by the prime
minister on October 2, 1959, in Nagaur district. Rajasthan was followed by Andhra Pradesh, which
also adopted the system in 1959. Thereafter, most of the states adopted the system.

73rd Constitutional Amendment Act, 1992 which came into force on 24 April 1993 gave
constitutional status to the Panchayati Raj Institutions. It has brought them under the purview of the
justiciable part of the Constitution. In other words, the state governments are under constitutional
obligation to adopt the new Panchayati raj system in accordance with the provisions of the act.

Consequently, neither the formation of panchayats nor the holding of elections at regular intervals
depending on the will of the state government anymore.

48. Consider the following statements regarding Legislative Council:


1. The creation of the legislative council of states needs a constitutional amendment.

2. It enjoys equal powers as the legislative assembly for ordinary bills.

Which of the statements given above is/are correct?

A. 1 only
B. 2 only
C. Both 1 and 2
D. Neither 1 nor 2
Explanation: The Parliament can abolish a legislative council or create it if the legislative assembly of
the concerned state passes a resolution to that effect. Such a specific resolution must be passed by
the state assembly by a special majority. This Act of Parliament is not to be deemed as an amendment
of the Constitution for the purposes of Article 368 and is passed as an ordinary piece of legislation
by a simple majority.

An ordinary bill can originate in either House of the state legislature. If the assembly rejects the
amendments suggested by the council or the council rejects the bill altogether or does not take any
action for three months, then the assembly may pass the bill again and transmit the same to the
council. If the council rejects the bill again or passes the bill with amendments not acceptable to the
assembly or does not pass the bill within one month, then the bill is deemed to have been passed by
both the Houses in the form in which it was passed by the assembly for the second time. Therefore,
the ultimate power of passing an ordinary bill is vested in the assembly. At most, the council can
detain or delay the bill for a period of four months—three months in the first instance and one month
in the second instance. The Constitution does not provide for the mechanism of a joint sitting of both
the Houses.

49. Under which of the following circumstances the members of the State Legislative Assembly is/are
disqualified?
1. if he is an undischarged insolvent.

2. if he is of unsound mind and stands so declared by a court

3. If he is disqualified under the provisions of the Tenth schedule.

Select the correct answer using the code given below.

A. 1 and 2 only
B. 2 only
C. 1 and 3 only
D. 1, 2 and 3
Explanation: Under the Constitution, a person shall be disqualified for being chosen as and for being
a member of the legislative assembly:

• if he holds any office of profit under the Union or State government (except that of a minister
or any other office exempted by state legislature).
• if he is of unsound mind and stands so declared by a court. Hence, statement 2 is correct.
• if he is an undischarged insolvent. Hence, statement 1 is correct.
• if he is not a citizen of India or has voluntarily acquired the citizenship of a foreign state or is
under any acknowledgement of allegiance to a foreign state and
• if he is so disqualified under any law made by Parliament

The constitution also lays down that a person shall be disqualified for being a member of either
House of state legislature if he is so disqualified on the ground of defection under the provisions of
the Tenth schedule.

If one is found guilty of an illegal practice in relation to election and being a director or managing
agent of a corporation in which government has a financial interest. Section 8 of the RP Act deals
with disqualification on conviction for certain offences: A person convicted of any offence and
sentenced to imprisonment for varying terms under Sections 8 (1) (2) and (3) shall be disqualified
from the date of conviction and shall continue to be disqualified for a further period of six years since
his release.

In 2015, Supreme Court held that even after a Returning Officer has declared the result, the election
can be nullified if candidate has not disclosed criminal records.

50. Which of the following committees made recommendations with respect to Centre- State relations?
1. Sarkaria Commission

2. M M Punchhi Commission

3. Rajamannar Committee

Select the correct answer using the code given below.

A. 1 and 2 only
B. 2 and 3 only
C. 1 and 3 only
D. 1, 2 and 3

Explanation: Justice R. S. Sarkaria Commission was appointed in June 1983. It stressed cooperative
federalism and noted that federalism is more a functional arrangement for cooperative action than
a static institutional concept. At the same time, it did not equate strong center with centralization of
powers because over-centralization leads to blood pressure at the center and anemia at the
periphery.

M M Punchhi Commission was set up in 2007 and it gave its recommendations in 2010. Some of the
important recommendations are as follows:

• There should be a consultation process between unions and states via Inter-state Council for
legislation on concurrent subjects.
• Regarding state bills, the President’s pocket veto should end and there should be a reasonable
time (6 months) in which President communicates his decision.
• The treaty-making powers of union should be regulated and states should get greater
participation in treaties where the interests of states are involved.
• Governor should get clear guidelines for the appointment of Chief Ministers so that he does
not misuse his discretionary powers in this context. etc.
Rajmannar committee was set in 1969. Apart from making a call for immediate constitution of the
Inter-state Council, this committee made the following recommendations:

• Union government should not take any decision without consulting the inter-state council
when such a decision can affect the interests of one or more states.
• Every bill which affects the interests of the states should be first referred to the inter-state
council before it is introduced in parliament.
• Article 356 should be used only in rare cases of a complete breakdown of law and order in
the state. etc.

51. Which of the following is/are correct regarding impeachment of the President?
1. He can be impeached only on the charge of violation of Constitution.

2. On removal of President, Vice-President takes charge as president for the remaining tenure.

Select the correct answer using the code given below.

A. 1 only
B. 2 only
C. Both 1 and 2
D. Neither 1 nor 2
Explanation: Article 56(1)(b) states that President may, for violation of the constitution, be removed
from office by impeachment. However, Constitution does not give description of what amounts to
violation of constitution.

The nominated members of either House of Parliament can participate in the impeachment of the
President though they do not participate in his election. Unlike in USA, Vice president does not take
charge for remaining tenure but only till the new president is elected. The impeachment charges can
be initiated by either House of Parliament. These charges should be signed by one-fourth members
of the House (that framed the charges), and a 14 days’ notice should be given to the President. After
the impeachment resolution is passed by a majority of two-thirds of the total membership of that
House, it is sent to the other House, which should investigate the charges. If the other House also
sustains the charges and passes the impeachment resolution by a majority of two-thirds of the total
membership, then the President stands removed from his office.
52. Consider the following statements regarding Cabinet Committees:
1. They are extra constitutional bodies.

2. All cabinet committees are headed by the Prime minister.

Which of the statements given above is/are correct?

A. 1 only
B. 2 only
C. Both 1 and 2
D. Neither 1 nor 2
Explanation: The following are the features of Cabinet Committees:

• They are extra-constitutional in emergence. In other words, they are not mentioned in the
Constitution. However, the Rules of Business provide for their establishment.
• They are of two types—standing and ad hoc. The former are of a permanent nature while the
latter are of a temporary nature. The ad hoc committees are constituted from time to time to
deal with special problems. They are disbanded after their task is completed.
• They are set up by the Prime Minister according to the exigencies of the time and requirements
of the situation. Hence, their number, nomenclature, and composition varies from time to
time.
• Their membership varies from three to eight. They usually include only Cabinet Ministers.
However, the non-cabinet Ministers are not debarred from membership. They not only include
the Ministers in charge of subjects covered by them but also other senior Ministers.
• They are mostly headed by the Prime Minister. Sometimes other Cabinet Ministers, particularly
the Home Minister or the Finance Minister, also acts as their Chairman. But, in case the Prime
Minister is a member of a committee, he invariably presides over it.

53. With reference to political parties as recognized by the Election Commission (EC), consider the following
statements:
1. EC recognizes parties as national parties or state parties on the basis of their poll performance.

2. The recognition granted by EC to political parties determines their right to certain privileges in
elections.

Which of the statements given above is/are correct?

A. 1 only
B. 2 only
C. Both 1 and 2
D. Neither 1 nor 2
Explanation: Article 324 of the Indian Constitution states that the superintendence, direction, and
control of the preparation of the electoral rolls for, and the conduct of, all elections to Parliament
and to the Legislature of every State and of elections to the offices of President and Vice President
held under this Constitution shall be vested in the Election Commission of India (ECI).

The Election Commission registers political parties for the purpose of elections and grants them
recognition as national or state parties on the basis of their poll performance. The other parties are
simply declared as registered unrecognized parties.

The recognition granted by the Commission to the parties determines their right to certain privileges
like allocation of the party symbols, provision of time for political broadcasts on the state-owned
television and radio stations, and access to electoral rolls.

However, the ECI is not empowered to de-register parties on the grounds of violating the
Constitution or breaching the undertaking given to it at the time of registration. A party can only be
de-registered if its registration was obtained by fraud; if it is declared illegal by the Central
Government; or if a party amends its internal Constitution and notifies the ECI that it can no longer
abide by the Indian Constitution.

54. Consider the following statements regarding the First Past the Post system of elections in India:
1. In this system each constituency elects one representative.

2. The winning candidate in this system must secure more than fifty percent votes.

Which of the statements given above is/are correct?

A. 1 only
B. 2 only
C. Both 1 and 2
D. Neither 1 nor 2
Explanation: In the First Past the Post System of elections, the country is divided into small
geographical units called constituencies. Every constituency elects one representative.

The winning candidate need not secure a majority of the votes i.e. (50%+1) votes. In the electoral
race, the candidate who is ahead of others, who crosses the winning post first of all, is the winner.This
method is also called the Plurality System. It is followed in India & United Kingdom.

55. With reference to Inter-state council, which of the following statements is/are correct?
1. It was established on the recommendations of the Punchhi Commission.

2. Its secretariat also functions as the secretariat of the Zonal Councils.


Select the correct answer using the code given below.

A. 1 only
B. 2 only
C. Both 1 and 2
D. Neither 1 nor 2
Explanation: In pursuance of the recommendations of the Sarkaria Commission, the Janata Dal
Government headed by V. P. Singh established the Inter-State Council in 1990.

The Council is assisted by a secretariat called the Inter-State Council Secretariat. This secretariat was
set up in 1991 and is headed by a secretary to the Government of India. Since 2011, it is also
functioning as the secretariat of the Zonal Councils.

56. Consider the following statements regarding the Organisation of Islamic Cooperation (OIC):
1. Its membership is spread over four continents.

2. Recently, India was granted the status of an Observer Member.

Which of the statements given above is/are correct?

A. 1 only
B. 2 only
C. Both 1 and 2
D. Neither 1 nor 2
Explanation: The Organisation of Islamic Cooperation (OIC) is the second-largest organization after
the United Nations with a membership of 57 states spread over four continents (Africa, Asia, Europe
and South America). The Organization is the collective voice of the Muslim world. It endeavours to
safeguard and protect the interests of the Muslim world in the spirit of promoting international
peace and harmony among various people of the world.

In 2019, India made its maiden appearance at the OIC Foreign Ministers’ meeting, as a “guest of
honour”. India does not have an Observer Status of the OIC.

57. With reference to the Constitution of India, consider the following pairs:
Subject List

1. Forests Union List

2. Agriculture State List

3. Education Concurrent List


Which of the pairs given above is/are correctly matched?

A. 1 and 3 only
B. 2 only
C. 2 and 3 only
D. 1, 2 and 3
Explanation: Union List includes subjects like, War and Peace, Naval, military and air force works,
Foreign jurisdiction., Citizenship, naturalization and aliens. etc.

State List includes subjects like Agriculture, Police, Prison, Local Government, Public Health, Land,
Liquor, etc.

Concurrent List includes subjects like Education, Transfer of Property other than Agricultural land,
Forests, Trade Unions, etc.

Residuary Powers include all other matters not mentioned in any of the lists such as Cyber Laws.
Union Legislature alone has the power to legislate on such matters.

Through the 42nd Amendment Act of 1976, five subjects were transferred from State to Concurrent
List. They are:

• Education
• Forests
• Weights & Measures
• Protection of Wild Animals and Birds
• Administration of Justice

58. With reference to the Municipal Corporations, consider the following statements:
1. They are established in the states by an act of the concerned state legislatures.

2. The Mayor is the chief executive authority of the corporation.

Which of the statements given above is/are correct?

A. 1 only
B. 2 only
C. Both 1 and 2
D. Neither 1 nor 2
Explanation: Municipal corporations are created for the administration of big cities like Delhi,
Mumbai, Kolkata, Hyderabad, Bangalore, and others. They are established in the states by the acts
of the concerned state legislatures, and in the union territories by the acts of the Parliament of India.
There may be one common act for all the municipal corporations in a state or a separate act for each
municipal corporation.
A municipal corporation has three authorities, namely,

• the council,
• the standing committees and
• the commissioner.
The Council is headed by a Mayor. He is assisted by a Deputy Mayor. He is elected in a majority of
the states for a one-year renewable term. He is basically an ornamental figure and a formal head of
the corporation. His main function is to preside over the meetings of the Council.

The Municipal Commissioner is responsible for the implementation of the decisions taken by the
council and its standing committees. Thus, he is the chief executive authority of the corporation. He
is appointed by the state government and is generally a member of the IAS.

59. Indian federalism differs from that of the federalism of the United States of America. In this context,
consider the differences between the federal or supreme courts of India and the USA:
1. The original jurisdiction of the American Supreme Court is wider than that of the Indian Supreme
Court.

2. The appellate jurisdiction of both the courts is confined to constitutional cases only.

3. The Indian Supreme Court has advisory jurisdiction whereas the American Supreme Court has no
advisory jurisdiction.

Which of the statements given above is/are correct?

A. 2 only
B. 1 and 3 only
C. 1, 2 and 3
D. 2 and 3 only
Explanation:
60. With reference to co-operative societies, consider the following statements:
1. Elections to a co-operative society are conducted by State Election Commission.

2. Number of directors of a co-operative society shall not exceed twenty-one.

3. Cooperative Board shall have at least two women representatives.

Which of the statements given above is/are correct?

A. 1 and 2 only
B. 2 and 3 only
C. 1 and 3 only
D. 1, 2 and 3
Explanation: The superintendence, direction, and control of the preparation of electoral, rolls and
the conduct of elections to a co-operative society shall vest in such body, as may be provided by the
state legislature.

The board shall consist of such number of directors as may be provided by the state legislature. But,
the maximum number of directors of a co-operative society shall not exceed twenty-one.

The state legislature shall provide for the reservation of one seat for the Scheduled Castes or the
Scheduled Tribes and two seats for women on the board of every co-operative society having
members from such a category of persons.

61. Supreme Court is a Court of Record. It implies that


1. The judgements of Supreme Court are recorded for perpetual memory.

2. The Supreme Court has the power to punish for contempt of court.

Select the correct answer using the code given below.

A. 1 only
B. 2 only
C. Both 1 and 2
D. Neither 1 nor 2
Explanation: As a Court of Record, the Supreme Court has two powers:

• The judgements, proceedings and acts of the Supreme Court are recorded for perpetual
memory and testimony. These records are admitted to be of evidentiary value and cannot be
questioned when produced before any court. They are recognised as legal precedents and legal
references.
• It has power to punish for contempt of court, either with simple imprisonment for a term up
to six months or with fine up to Rs.2,000 or with both. In 1991, the Supreme Court has ruled
that it has power to punish for contempt not only of itself but also of high courts, subordinate
courts and tribunals functioning in the entire country.

62. The Eleventh Schedule of the Constitution places which of the following under the purview of
panchayats?
1. Agriculture

2. Khadi, village and cottage industries

3. Slum improvement and upgradation

4. Public Distribution System

Select the correct answer using the code given below.

A. 1 and 2 only
B. 3 and 4 only
C. 1, 2 and 4 only
D. 2, 3 and 4 only
Explanation: Eleventh Schedule contains the following 29 functional items placed within the purview
of panchayats:

• Agriculture, including agricultural extension


• Land improvement, implementation of land reforms, land consolidation, and soil conservation
• Minor irrigation, water management, and watershed development
• Animal husbandry, dairying, and poultry
o Fisheries
• Social forestry and farm forestry
• Minor forest produce
• Small-scale industries, including food processing industries
• Khadi, village, and cottage industries
• Rural housing
• Drinking water
• Fuel and fodder
• Roads, culverts, bridges, ferries, waterways, and other means of communication
• Rural electrification, including distribution of electricity
• Non-conventional energy sources
• Poverty alleviation programme
• Education, including primary and secondary schools
• Technical training and vocational education
• Adult and non-formal education
• Libraries
• Cultural activities
• Markets and fairs
• Health and sanitation including hospitals, primary health centers, and dispensaries
• Family welfare
• Women and child development
• Social welfare, including the welfare of the handicapped and mentally retarded
• Welfare of the weaker sections, and in particular, of the scheduled castes and the scheduled
tribes
• Public distribution system
• Maintenance of community assets.
Slum improvement and up-gradation are under the Twelfth Schedule of the Constitution.

63. Which of the following is/are not compulsory provisions under the 73 rd Constitutional Amendment Act
of 1992?
1. Organization of Gram Sabha in a village or group of villages

2. Constitution of a State Finance Commission

3. Giving representation to members of the Parliament in the panchayats

Select the correct answer using the code given below.

A. 1 and 2 only
B. 3 only
C. 1 and 3 only
D. 2 only
Explanation: The 73rd Constitutional Amendment Act of 1992 provides for some compulsory and
some voluntary provisions. These are:

A. Compulsory Provisions

• Organization of Gram Sabha in a village or group of villages. Hence option 1 is not correct.
• Establishment of panchayats at the village, intermediate, and district levels.
• Direct elections to all seats in panchayats at the village, intermediate, and district levels.
• Indirect elections to the post of chairperson of panchayats at the intermediate and district
levels.
• 21 years to be the minimum age for contesting elections to panchayats.
• Reservation of seats (both members and chairpersons) for SCs and STs in panchayats at all
three levels.
• Reservation of one-third of seats (both members and chairpersons) for women in Panchayats
at all three levels.
• Fixing tenure of five years for panchayats at all levels and holding fresh elections within six
months in the event of supersession of any panchayat.
• Establishment of a State Election Commission for conducting elections to the panchayats.
• Constitution of a State Finance Commission after every five years to review the financial
position of the panchayats.

B. Voluntary Provisions

• Giving representation to members of the Parliament (both the Houses) and the state
legislature (both the Houses) in the panchayats at different levels falling within their
constituencies.
• Providing reservation of seats (both members and chairpersons) for backward classes in
Panchayats at any level.
• Granting powers and authority to the panchayats to enable them to function as institutions of
self-government (in brief, making them autonomous bodies).
• Devolution of powers and responsibilities upon panchayats to prepare plans for economic
development and social justice; and to perform some or all of the 29 functions listed in the
Eleventh Schedule of the Constitution.
• Granting financial powers to the panchayats, that is, authorizing them to levy, collect, and
appropriate taxes, duties, tolls, and fees.

64. Consider the following statements regarding the parliamentary privileges available to the members:

1. They cannot be arrested during the session of Parliament for both civil and criminal offences.
2. The presiding officer of the house has the final authority in deciding matters of breach of privileges.

Which of the statements given above is/are correct?

A. 1 only
B. 2 only
C. Both 1 and 2
D. Neither 1 nor 2
Explanation: The Constitution (Article 105) expressly mentioned two privileges, that is, freedom of
speech in Parliament and right of publication of its proceedings.

According to Article 105

• Subject to the provisions of this Constitution and to the rules and standing orders regulating
the procedure of Parliament, there shall be freedom of speech in Parliament.
• No member of Parliament shall be liable to any proceedings in any court in respect of anything
said or any vote given by him in Parliament or any committee thereof, and no person shall be
so liable in respect of the publication by or under the authority of either House of Parliament
of any report, paper, votes or proceedings.
Contempt of House by an individual is not a parliamentary privilege mentioned in the Constitution.

65. Which of the following is not correct when Vice-President is discharging functions of President?
A. He enjoys all immunities which are available to the President for his official acts.
B. He continues to perform the duties of chairman of Rajya Sabha.
C. He can pardon the sentence of a court martial.
D. He can promulgate ordinances on the advice of Council of Ministers.
Explanation: When Vice-President is acting as President, he enjoys all the powers and Privileges of
the President. While acting as President or discharging the functions of President, the Vice-President
does not perform the duties of the office of the chairman of Rajya Sabha. During this period, those
duties are performed by the Deputy Chairman of Rajya Sabha.

66. Consider the following statements with reference to The Provisions of the Panchayats (Extension to
Scheduled Areas) Act, 1996 (PESA):
1. The provisions of the Act apply to the states of the North East India only.

2. Ministry of Rural Development and Ministry of Development of North Eastern Region are the nodal
ministries for the implementation of the provisions under this Act.

Which of the statements given above is/are correct?

A. 1 only
B. 2 only
C. Both 1 and 2
D. Neither 1 nor 2
Explanation: PESA Act or The Provisions of the Panchayats (Extension to Scheduled Areas) Act, 1996
is called a ‘Constitution within the Constitution’ as it extends the Provision of Panchayati Raj (Part IX)
of the Constitution to the Fifth Schedule areas of 10 States under clause (1) of Article 244 with certain
modifications and exceptions. The 10 states: Andhra Pradesh, Chhattisgarh, Gujarat, Himachal
Pradesh, Jharkhand, Madhya Pradesh, Maharashtra, Orissa, Rajasthan, and Telangana.

The Ministry of Panchayati Raj is the nodal Ministry for the implementation of the provisions of the
PESA Act.

67. Consider the following statements regarding the United Nations Refugee Convention, 1951:
1. It is based on the core principle of non-refoulment, which asserts that a refugee should not be
returned to a country where they face serious threats to their life or freedom.

2. It has not clearly defined the term Refugee and anyone can declare him/herself as a refugee without
any verification.

3. India ratified the Convention in 2010.

Which of the statements given above is/are correct?

A. 1 only
B. 2 and 3 only
C. 2 only
D. 1, 2 and 3
Explanation: Though India has not signed the United Nations Refugee Convention, 1951 but the
refugees and asylum seekers were entitled to the rights in Articles 14, 20 and 21 of the Constitution.

The Convention Relating to the Status of Refugees, also known as the 1951 Refugee Convention or
the Geneva Convention of 28 July 1951, is a United Nations multilateral treaty that defines who a
refugee is, and sets out the rights of individuals who are granted asylum and the responsibilities of
nations that grant asylum.

The core principle is non-refoulement, which asserts that a refugee should not be returned to a
country where they face serious threats to their life or freedom. This is now considered a rule of
customary international law.

India is not party to the 1951 Refugee Convention or its 1967 Protocol and does not have a national
refugee protection framework. However, it continues to grant asylum to a large number of refugees
from neighbouring States and respects UNHCR's mandate for other nationals, mainly from
Afghanistan and Myanmar.

68. The Model Code of Conduct comes immediately into effect:


A. once the Election Commission announces the schedule of elections.
B. once the notification calling electorate to elect members of a house is issued.
C. always at the first date of the month in which elections are to be conducted.
D. once the candidates start filing their nominations in the constituencies.
Explanation: The Commission normally announces the schedule of elections in a major press
conference a few weeks before the formal process is set in motion. The Model Code of Conduct for
the guidance of candidates and political parties comes immediately into effect after such
announcement.

The formal process for the elections starts with the Notification or Notifications calling upon the
electorate to elect Members of a House. As soon as Notifications are issued, candidates can start
filing their nominations in the constituencies from where they wish to contest.

69. With reference to Ayushman Bharat- Pradhan Mantri Jan Arogya Yojana (PMJAY), consider the following
statements:
1. It is the world’s largest government- funded healthcare program.

2. The beneficiaries under the scheme will be identified on the basis of the Socio- Economic Caste
Census, 2011.

3. National Health Authority (NHA) is the nodal agency for its implementation.

Which of the statements given above are correct?

A. 1 and 2 only
B. 2 and 3 only
C. 1 and 3 only
D. 1, 2 and 3
Explanation: AB PMJAY is a centrally sponsored health assurance scheme. It provides a cover of up
to Rs. 5 lakhs per family per year for secondary and tertiary care hospitalization. This is the “world’s
largest government-funded healthcare program” targeting more than 50 crore beneficiaries. It
provides cashless and paperless access to services for beneficiaries at point of service. Beneficiary
families have been identified from Socio-Economic Caste Census, 2011.

National Health Authority (NHA) is the apex body responsible for implementing India’s flagship public
health insurance/assurance scheme called “Ayushman Bharat Pradhan Mantri Jan Arogya Yojana” &
has been entrusted with the role of designing strategy, building technological infrastructure, and
implementing of “National Digital Health Mission” to create a National Digital Health Ecosystem.

70. Which of the following is not a function of the Central Bureau of Investigation (CBI)?
A. Investigating cases of corruption, bribery, and misconduct of Central government employees.
B. Probe terror attacks including bomb blasts, hijacking of aircraft and ships, attacks on nuclear
installations, and use of weapons of mass destruction.
C. Investigating serious crimes, having national and international ramifications, committed by
organized gangs of professional criminals.
D. Maintaining crime statistics and disseminating criminal information.
Explanation: The Central Bureau of Investigation (CBI) was set up in 1963 by a resolution of the
Ministry of Home Affairs. Later, it was transferred to the Ministry of Personnel and now it enjoys the
status of an attached office. The Special Police Establishment (which looked into vigilance cases)
setup in 1941 was also merged with the CBI. The establishment of the CBI was recommended by the
Santhanam Committee on Prevention of Corruption (1962–1964). The CBI is not a statutory body. It
derives its powers from the Delhi Special Police Establishment Act, of 1946.

The functions of CBI are:

• Investigating cases of corruption, bribery, and misconduct of Central government employees.


• Investigating cases relating to infringement of fiscal and economic laws, that is, breach of laws
concerning export and import control, customs and central excise, income tax, foreign
exchange regulations, and so on. However, such cases are taken up either in consultation with
or at the request of the department concerned.
• Investigating serious crimes, having national and international ramifications, committed by
organized gangs of professional criminals.
• Coordinating the activities of the anticorruption agencies and the various state police forces
• Taking up, on the request of a state government, any case of public importance for
investigation.
• Maintaining crime statistics and disseminating criminal information.
Probing terror attacks including bomb blasts, hijacking of aircraft and ships, attacks on nuclear
installations, and use of weapons of mass destruction is the function of the National Investigation
Agency.

71. Which of the following statements is/are correct with regard to the consequences of the proclamation
of a Financial Emergency?
1. Centre acquires full control over the states in financial matters.
2. President may reserve all money bills or other financial bills for consideration after they are passed
by the legislature of the state.

3. President may issue directions for the reduction of salaries and allowances of the judges of the
Supreme Court and the high court.

Select the correct answer using the code given below.

A. 1 only
B. 1 and 2 only
C. 2 and 3 only
D. 1, 2 and 3
Explanation: Article 360 empowers the president to proclaim a Financial Emergency if he is satisfied
that a situation has arisen due to which the financial stability or credit of India or any part of its
territory is threatened.

The consequences of the proclamation of a Financial Emergency are as follows:

• The executive authority of the Centre extends to the giving of directions to any state to observe
such canons of financial propriety as may be specified in the directions, andsuch other
directions to any state as the President may deem necessary and adequate for the purpose.
• Any such direction may include a provision requiring the reduction of salaries and allowances
of all or any class of persons serving in the state; and the reservation of all money bills or other
financial bills for the consideration of the President after they are passed by the legislature of
the state. Hence statement 2 is correct.
• The President may issue directions for the reduction of salaries and allowances of all or any
class of persons serving the Union; and the judges of the Supreme Court and the high court.
Thus, during the operation of a financial emergency, the Centre acquires full control over the states
in financial matters.

72. Which of the following are the unitary features of Indian Constitution?
1. Single constitution

2. Emergency Provisions

3. All-India Services

4. Integrated Election Machinery

5. Appointment of Governor

Select the correct answer using the code given below.

A. 1, 2, 3 and 4 only
B. 2, 4 and 5 only
C. 1, 3, 4 and 5 only
D. 1, 2, 3, 4 and 5
Explanation: Unitary state is a system of political organization in which most or all of the governing
power resides in a centralized government, in contrast to a federal state.

Despite being a federation Indian constitution possesses many unitary features such as

• Single Constitution
• Emergency Provisions
• All-India Services
• Appointment of Governor
• Integrated Election Machinery
• Veto Over State Bills
• Parliament’s Authority Over State List
• Integrated Audit Machinery
• Integrated Judiciary
• No Equality of State Representation

73. Which of the following services, at present, are regulated under Information Technology (Intermediary
Guidelines and Digital Media Ethics Code) Rules, 2021?
1. Social media sites

2. Over-the-top (OTT) streaming services

3. Online gaming services

4. Digital news sites

Select the correct answer using the code given below.

A. 1 and 2 only
B. 3 and 4 only
C. 1, 2 and 4 only
D. 1, 2, 3 and 4
Explanation: Information Technology (Intermediary Guidelines and Digital Media Ethics Code) Rules
2021 broadly deal with Social media sites, Digital news sites, and Over-the-top (OTT) streaming
platforms.

It covers guidelines on Social Media Intermediaries, Grievance Appellate Committees, ensuring


online safety & dignity of users, and educating users about Privacy Policies. The Ministry of
Information and Broadcasting is responsible for the formulation of an oversight mechanism to ensure
the same.
74. With reference to 'constitutional torts' in India, consider the following statements:
1. In India, there is no legislation, which governs the liability of the State for the torts committed by its
servants.

2. A court of law can award monetary compensation to the victim in such cases.

Which of the statements given above is/are correct?

A. 1 only
B. 2 only
C. Both 1 and 2
D. Neither 1 nor 2

Explanation: A ‘constitutional tort’ is a violation of one’s constitutional rights, particularly


fundamental rights, by an agent of the government, acting in his/her official capacity.

In India, there is no legislation, which governs the liability of the State for the torts committed by its
servants. It is Article 300 of the Constitution of India, which enumerates the liability of the Union or
State in the tortious act of the Government.

A court of law can award monetary compensation to the victim in such a case. for eg: in Supreme
Court in State of Rajasthan V. Mst. Vidyawati Case, the Court ordered the defendant, i.e., the State
of Rajasthan to pay compensation of Rs. 15,000 to the plaintiff.

75. Global Risks Report 2023, an annual report based on Global Risks Perception Survey was released by
A. World Bank
B. World Economic Forum
C. United Nations Office for Disaster Risk Reduction
D. Germanwatch
Explanation: The World Economic Forum has published the Global Risks Report 2023. The report is
published annually based on Global Risks Perception Survey.

It highlights key risks across five categories: Economic, Environmental, Geopolitical, Societal, and
Technological. Global risk is defined as the possibility of the occurrence of an event or condition that,
if it occurs, would negatively impact a significant proportion of global GDP, population, or natural
resources.

Key findings:

• The top 5 risks for India are Digital inequality, Geopolitical contest for resources, Cost-of-living
crisis, Debt crises, Natural disasters, and Extreme weather events over the short and medium
term.
• The cost of living dominates global risks in the next 2 years while climate action failure
dominates the next decade.
• The economic effects of COVID-19 and the Ukraine war resulted in rising inflation, rapid
normalization of monetary policies, low-growth, low-investment era.
• Technology such as AI, quantum computing and biotechnology, etc. will exacerbate
inequalities and the digital divide.

76. The PRANA portal was continuously in the news in recent times. Which of the following statements best
explains its objective?
A. It is a portal for monitoring of implementation of National Clean Air Programme.
B. It will provide online education to far flung areas where physical infrastructure of education is not
present.
C. It will provide the Gram Panchayats with a single interface to prepare and implement their Gram
Panchayat Development Plan.
D. It will ensure hassle-free disbursement of pension to retired state government employees.
Explanation: PRANA (Portal for Regulation of Air-pollution in Non-Attainment cities) is a portal for
monitoring of the implementation of the National Clean Air Programme (NCAP).

It will support tracking of physical as well as financial status of city air action plan implementation
and disseminate information on air quality management efforts under NCAP to public.

It would also disseminate information on air quality to the public while also providing comprehensive
information related to NCAP programme details, progress, city action plans, implementation updates
by city/state/ national level agencies, air quality data, and trends, etc.

77. Consider the following statements with respect to the office of Lokayuktas:
1. The institution of Lokayukta was established for the first time in Maharashtra in 1971.

2. The Lokayukta of a state is always appointed by the chief justice of the high court of the concerned
state.

3. The recommendations made by the Lokayukta are only advisory and not binding on the state
government.

Which of the statements given above is/are correct?

A. 2 and 3 only
B. 1 only
C. 1, 2 and 3
D. 1 and 3 only
Explanation: The Lokayukta is the Indian Parliamentary Ombudsman, executed into power, through
and for, each of the State Governments of India. It is brought into effect in a state, after passing the
Lokayukta Act in the respective state Legislature and a person of reputable background is nominated
to the post. The post is created to quickly address grievances against the working integrity and
efficiency of the government or its administration (public servants). Once appointed, Lokayukta
cannot be dismissed or transferred by the government, and can only be removed by passing an
impeachment motion by the state assembly, making it a powerful deterrent against corruption and
maladministration of the governing system.

Even much before the enactment of the Lokpal and Lokayuktas Act (2013) itself, many states had
already set up the institution of Lokayuktas. It must be noted here that the institution of lokayukta
was established first in Maharashtra in 1971. Although Odisha had passed the Act in this regard in
1970, it came into force only in 1983.

The lokayukta and upalokayukta are appointed by the governor of the state. While appointing, the
governor in most of the states (varies from state to state) consults

• the chief justice of the state high court, and


• the leader of the Opposition in the state legislative assembly.
• the speaker of the legislative assembly (optional)
• In case the state has a legislative council, the chairman of the legislative council and the leader
of the opposition of the legislative council are also consulted
The recommendations made by the Lokayukta are only advisory and not binding on the state
government.

78. Consider the following statements regarding the Pradhan Mantri Garib Kalyan Anna Yojana (PMGKAY):
1. It is an integrated food security scheme for providing free foodgrains to Antodaya Ann Yojna (AAY)
and Primary Household (PHH) beneficiaries under National Food Security Act (NFSA), 2013

2. The beneficiaries will also be getting food grains at subsidized prices.

Which of the statements given above is/are correct?

A. 1 only
B. 2 only
C. Both 1 and 2
D. Neither 1 nor 2
Explanation: Centre named its integrated food security scheme for providing free foodgrains to
Antodaya Ann Yojna (AAY) and Primary Household (PHH) beneficiaries under National Food Security
Act (NFSA), 2013 as Pradhan Mantri Garib Kalyan Anna Yojana (PMGKAY).
It will strengthen provisions of NFSA, 2013 in terms of accessibility, affordability, and availability of
food grains for the poor. It will subsume two subsidy schemes of the Department of Food & Public
Distribution

• Food Subsidy to FCI: It is in form of a consumer subsidy (Difference between Economic cost incurred
by FCI and realized Central Issue Price) and a buffer subsidy (Cost of holding and maintaining buffer
stock).

• Food Subsidy for decentralized procurement states dealing with procurement, allocation, and
delivery of free foodgrains to states under NFSA.

Earlier, a scheme with a similar name (PMGKAY) was implemented during the Covid-19 pandemic.

Difference between the two schemes: Earlier, NFSA beneficiaries were getting foodgrains
entitlement (35 kg per AAY Household and 5 kg per person to PHH) at a subsidized rates (Rs 3/kg
rice, Rs 2/kg wheat, and Rs 1/kg coarse grains). Now, the government has done away with subsided
prices and providing food grains free of cost for a year. However, an additional quantity of 5 kg of
food grains will not be provided.

79. Food Safety and Standards Authority of India (FSSAI) falls under which of the following ministries?
A. Ministry of Health & Family Welfare
B. Ministry of Food Processing Industries
C. Ministry of Commerce and Industry
D. It is an independent body and not part of any ministry
Explanation: FSSAI is under Ministry of Health & Family Welfare. It is an autonomous statutory body
established under Food Safety and Standards Act (FSSA), 2006. FSSA 2006 consolidates various acts
& orders that had earlier handled food related issues in various Ministries.

It regulates and monitor, manufacture, processing, distribution of food while ensuring safe and
wholesome food to consumers.

Headquarters - Delhi

80. Which of the following statements is/are correct regarding Cabinet?


1. It is a smaller body than the Council of Ministers.

2. Its powers and functions are defined in the constitution.

3. It is an advisory body to the president and its advice is binding on him.

Select the correct answer using the code given below.


A. 1 only
B. 1 and 3 only
C. 2 and 3 only
D. 1 and 2 only
Explanation: It is a smaller body consisting of 15 to 20 ministers. Thus, it is a part of the Council of
Ministers. It was included in the Constitution by the 44th Constitutional Amendment Act. Now also,
Article 352 only defines the cabinet as ‘the council consisting of the Prime Minister and other ministers
of cabinet rank appointed under Article 75’ and does not describe its powers and functions.

It is an advisory body to the President and its advice is binding on him/her. The 44 th Constitutional
Amendment Act provides that the President may send back the advice of the Cabinet for reconsideration
once. But the reconsidered advice is binding on the President.

81. Consider the following statements regarding the territorial extent of Central and State legislation in India:
1. The territorial jurisdiction of the Parliament is absolute in nature for whole or any part of the territory
of India.

2. The laws enacted by state legislature cannot be applicable outside the state.

Which of the statements given above is/are correct?

A. 1 only
B. 2 only
C. Both 1 and 2
D. Neither 1 nor 2
Explanation: The plenary power placed of the Parliament is not absolute in nature for whole or any part
of the territory of India. The Constitution places a number of restrictions on the same. For e.g. The
governor is empowered to direct that an act of Parliament does not apply to a scheduled area in the
state or apply with specified modifications and exceptions; The President can make regulations for the
peace, progress and good government of the four Union territories - the Andaman and Nicobar Islands,
Lakshadweep, Dadra and Nagar Haveli and Daman and Diu etc.

A state legislature can make laws for the whole or any part of the state. The laws made by a state
legislature are not applicable outside the state, except when there is a sufficient nexus between the
state and the object.

82. Consider the following statements with reference to zonal councils in India:
1. These are statutory bodies created under the States Reorganisation Act of 1956.

2. The Union Home Minister acts as the chairman of all the zonal councils.
Which of the statements given above is/are correct?

A. 1 only
B. 2 only
C. Both 1 and 2
D. Neither 1 nor 2
Explanation: The Zonal Councils are statutory (and not constitutional) bodies. They are established by
the States Reorganisation Act of 1956. The act divided the country into five zones (Northern, Central,
Eastern, Western and Southern) and provided a zonal council for each zone. However, the North-Eastern
Council was created by a separate Act of Parliament—the North-Eastern Council Act of 1971. The Union
Home Minister is the common chairman of all the zonal councils.

83. Which of the following statements is/are correct with respect to the High Courts of Union territories?
1. The High Court of Delhi was established by the Constitution under Article 239AA.

2. Establishing a High Court for a Union Territory would require a constitutional amendment.

Select the correct answer using the code given below.

A. 1 only
B. 2 only
C. Both 1 and 2
D. Neither 1 nor 2
Explanation: The Constitution, under Article 239AA, establishes the Union Territory of Delhi and its
legislature. It does not provide for High Court of Delhi. High Court of Delhi was established by enacting
the Delhi High Court Act, 1966.

Article 241 of the Constitution states that Parliament by law can establish High Court of any Union
Territory. Hence no Constitutional amendment is required.

84. Under the 73rd and 74th Constitutional Amendment Acts, no election to any municipality/panchayat can
be questioned except by an election petition presented to which of the following authority?
A. A district court in the district to which the municipality or the panchayat belongs
B. The High Court of the concerned state
C. Any authority as provided by the State legislature
D. The State Election Commission
Explanation: The 74th Constitutional Amendment Act of 1992 bars the interference by courts in the
electoral matters of municipalities. It declares that the validity of any law relating to the delimitation of
constituencies or the allotment of seats to such constituencies cannot be questioned in any court.
It further lays down that no election to any municipality is to be questioned except by an election petition
presented to such authority and in such a manner as provided by the state legislature.

The 73rd Constitutional Amendment Act of 1992 contains similar provisions in relation to the
panchayats.

85. Which of the following is/are the duties of the Indian Coast Guard?
1. Preservation and Protection of Marine Environment

2. Prevention and Control of Marine Pollution

3. Assisting the Customs and other authorities in anti-smuggling operations

Select the correct answer using the code given below.

A. 1 and 2 only
B. 2 only
C. 3 only
D. 1, 2 and 3
Explanation: The Indian Coast Guard is a maritime law enforcement and search and rescue agency of
India with jurisdiction over its territorial waters including its contiguous zone and exclusive economic
zone. The Indian Coast Guard was formally established on 1 February 1977 by the Coast Guard Act, 1978
of the Parliament of India.

Its duties are as follows:

• Safety and Protection of Artificial Islands and Offshore TerminalsProtection of Fishermen


• Assistance to Fishermen in Distress at Sea
• Preservation and Protection of Marine Environment
• Prevention and Control of Marine Pollution
• Assisting the Customs and other authorities in anti-smuggling operations
• Enforcement of Maritime Laws in Force
• Safety of Life and Property at Sea
• Collection of Scientific Data

86. With reference to the budgetary process in India, appropriation bill refers to:
A. authorization of the withdrawal from the Consolidated Fund of India.
B. taxation proposals of the government.
C. demands for grants by ministries.
D. provision to make any grant in advance for a part of the financial year.
Explanation: The Constitution states that ‘no money shall be withdrawn from the Consolidated Fund of
India except under appropriation made by law’. Accordingly, an appropriation bill is introduced to
provide for the appropriation, out of the Consolidated Fund of India. This act authorises (or legalises) the
payments from the Consolidated Fund of India. This means that the government cannot withdraw money
from the Consolidated Fund of India till the enactment of the appropriation bill.

87. Which of the following statements is/are correct with regard to discussions in the parliament?
1. During the Zero Hour, the members are free to raise any matter that they think is important.

2. During the Question Hour, Ministers are bound to give oral answers to all the types of questions.

Select the correct answer using code given below.

A. 1 only
B. 2 only
C. Both 1 and 2
D. Neither 1 nor 2
Explanation: Zero Hour is where members are free to raise any matter that they think is important but
the ministers are not bound to reply

During the Question Hour, which is held every day during the sessions of Parliament, Ministers respond
to questions raised by the members. However he is bound to give oral answers only to starred questions.
In case of unstarred questions, written replies will be given.

88. Consider the following statements regarding Council of Ministers:


1. The strength of the Council of Ministers cannot exceed 10 percent of total number of members of the
Lok Sabha.

2. Council of Ministers is collectively responsible to the Parliament.

Which of the statements given above is/are correct?

A. 1 only
B. 2 only
C. Both 1 and 2
D. Neither 1 nor 2
Explanation: Before the 91st Amendment Act (2003), the size of the Council of Ministers was determined
according to exigencies of time and requirements of the situation. But this led to very large size of the
Council of Ministers. Therefore, an amendment was made that the Council of Ministers shall not exceed
15 percent of total number of members of the House of People (or Assembly, in the case of the States).
The Council of Ministers is collectively responsible to the Lok Sabha.

89. 'The claims of the members of the Scheduled Castes and the Scheduled Tribes shall be taken into
consideration, consistently with the maintenance of efficiency of administration, in the making of
appointments to services and posts in connection with the affairs of the Union or a State'. This provision
was provided in the Indian Constitution under:
A. Part IV
B. Part III
C. Part XVI
D. Part XVII
Explanation: Apart from the Directives included in Part IV, there are some other Directives contained in other
parts of the Constitution. They are:

• Claims of SCs and STs to Services: The claims of the members of the Scheduled Castes and the
Scheduled Tribes shall be taken into consideration, consistently with the maintenance of efficiency
of administration, in the making of appointments to services and posts in connection with the affairs
of the Union or a State (Article 335 in Part XVI).
• Instruction in mother tongue: It shall be the endeavour of every state and every local authority within
the state to provide adequate facilities for instruction in the mother tongue at the primary stage of
education to children belonging to linguistic minority groups (Article 350-A in Part XVII).
• Development of the Hindi Language: It shall be the duty of the Union to promote the spread of the
Hindi language and to develop it so that it may serve as a medium of expression for all the elements
of the composite culture of India (Article 351 in Part XVII).
The above Directives are also non-justiciable in nature. However, they are also given equal importance and
attention by the judiciary on the ground that all parts of the constitution must be read together.

90. Which of the following are the features of fundamental rights?


1. They strike a balance between individual liberty and social control.

2. There are some fundamental rights that are available against the arbitrary action of private individuals.

3. There are some fundamental rights that are available only to the citizens.

4. All the fundamental rights are self-executory.

Select the correct answer using the code given below.

A. 1, 2 and 3 only
B. 2 and 4 only
C. 1 and 3 only
D. 1, 2, 3 and 4
Explanation: The Fundamental Rights guaranteed by the Constitution are characterized by the following:

• Some of them are available only to citizens while others are available to all persons whether citizens,
foreigners, or legal persons like corporations or companies.
• They are not absolute but qualified. The state can impose reasonable restrictions on them. However,
whether such restrictions are reasonable or not is to be decided by the courts. Thus, they strike a
balance between the rights of the individual and those of the society as a whole, between individual
liberty and social control
• All of them are available against the arbitrary action of the state. However, some of them are also
available against the action of private individuals.
• Some of them are negative in character, that is, place limitations on the authority of the State, while
others are positive in nature, conferring certain privileges on the persons.
• They are justiciable, allowing persons to move the courts for their enforcement, if and when they
are violated.
• They are defended and guaranteed by the Supreme Court. Hence, the aggrieved person can directly
go to the Supreme Court, not necessarily by way of appeal against the judgment of the high courts.
• They are not sacrosanct or permanent. The Parliament can curtail or repeal them but only by a
constitutional amendment act and not by an ordinary act. Moreover, this can be done without
affecting the ‘basic structure’ of the Constitution.
• They can be suspended during the operation of a National Emergency except for the rights
guaranteed by Articles 20 and 21. Further, the six rights guaranteed by Article 19 can be suspended
only when an emergency is declared on the grounds of war or external aggression (i.e., external
emergency) and not on the ground of armed rebellion (i.e., internal emergency).
• Their scope of operation is limited by Article 31A (saving of laws providing for the acquisition of
estates, etc.), Article 31B (validation of certain acts and regulations included in the 9th Schedule),
and Article 31C (saving of laws giving effect to certain directive principles).
• Their application to the members of armed forces, para-military forces, police forces, intelligence
agencies, and analogous services can be restricted or abrogated by the Parliament (Article 33).
• Their application can be restricted while martial law is in force in any area. Martial law means
‘military rule’ imposed under abnormal circumstances to restore order (Article 34). It is different
from the imposition of a national emergency.
• Most of them are directly enforceable (self-executory) while a few of them can be enforced on the
basis of a law made for giving effect to them. Such a law can be made only by the Parliament and
not by state legislatures so that uniformity throughout the country is maintained (Article 35).

91. Which of the following are the advantages of parliamentary democracy?


1. Responsible Government

2. Wide Representation

3. Strict Separation of powers

4. Harmony Between Legislature and Executive


Select the correct answer using the code given below.

A. 1, 2 and 3 only
B. 2, 3, and 4 only
C. 1 and 4 only
D. 1, 2 and 4 only
Explanation: Advantages of the Parliamentary System

• Harmony Between Legislature and Executive


• Responsible Government
• Ready Alternative Government
• Wide Representation
• Prevents Despotism.

92. Consider the following statements with respect to the Right to Equality:
1. Equality before the law is a positive concept which means the absence of any special privileges in
favour of any person.

2. Where equals and unequals are treated differently, Article 14 does not apply.

Which of the statements given above is/are correct?

A. 1 only
B. 2 only
C. Both 1 and 2
D. Neither 1 nor 2
Explanation:

Equality before Law and Equal Protection of Laws - Article 14 says that the State shall not deny to any person
equality before the law or equal protection of the laws within the territory of India. This provision confers
rights on all persons whether citizens or foreigners. Moreover, the word ‘person’ includes legal persons, viz,
statutory corporations, companies, registered societies or any other type of legal person.

The concept of ‘equality before the law’ (negative concept) is of British origin while the concept of ‘equal
protection of laws’ has been taken from the American Constitution. The first concept connotes:

• the absence of any special privileges in favour of any person,


• the equal subjection of all persons to the ordinary law of the land administered by ordinary law
courts, and
• no person (whether rich or poor, high or low, official or non-official) is above the law.

The second concept, on the other hand, connotes:


• the equality of treatment under equal circumstances, both in the privileges conferred and liabilities
imposed by the laws,
• the similar application of the same laws to all persons who are similarly situated, and
• the like should be treated alike without any discrimination.
Thus, the former is a negative concept while the latter is a positive concept. However, both of them aim
at establishing equality of legal status, opportunity and justice.

The Supreme Court held that where equals and unequals are treated differently, Article 14 does not
apply. While Article 14 forbids class legislation, it permits the reasonable classification of persons,
objects, and transactions by the law. But the classification should not be arbitrary, artificial, or evasive.
Rather, it should be based on an intelligible differential and substantial distinction.

93. Consider the following statements:


1. In the USA, a citizen by birth, as well as a naturalized citizen, is eligible for the office of President.

2. In India, only a citizen by birth and not a naturalized citizen is eligible for the office of President.

Which of the statements given above is/are not correct?

A. 1 only
B. 2 only
C. Both 1 and 2
D. Neither 1 nor 2
Explanation: One of the prima facie differences between the dual citizenship provisions is that the Indian
constitution actively prevents dual citizenship while the constitution of the United States does not
mention any restriction on it.

In India, both a citizen by birth as well as a naturalized citizen is eligible for the office of President, while
in the USA, only a citizen by birth and not a naturalized citizen is eligible for the office of President.

94. India was a pioneer when it comes to giving voting rights to the people. In this context, which of the
following countries gave universal adult franchises after India?
1. United States of America

2. Japan

3. France

Select the correct answer using the code given below.

A. 1 only
B. 1 and 3 only
C. 2 and 3 only
D. 1, 2 and 3
Explanation: In the nineteenth century struggles for democracy often centred around political equality,
freedom and justice. One major demand was the right for every adult citizen to vote. Many European
countries that were becoming more democratic did not initially allow all people to vote.In some
countries only people owning property had the right to vote. Often women did not have the right to
vote. In the United States of America, the blacks all over the country could not exercise the right to vote
until 1965.

India granted Universal Adult Franchise to its citizens in 1950, when the Constitution of India came into
force declaring India a democratic republic. It is provided under the Article 326 of the Indian Constitution.

France in 1944; Japan in 1945

95. Under the Constitution of India, which of the following rights and privileges are conferred on the citizens
of India and denied to aliens?
1. Right against discrimination on grounds of religion, race, caste, sex, or place of birth

2. Right to freedom of speech and expression, assembly, association, movement, residence, and
profession

3. Right to serve as a judge in the higher judiciary

4. Right to contest for membership of the Parliament and the state legislature

Select the correct answer using the code given below.

A. 2 and 3 only
B. 2 and 4 only
C. 1 and 3 only
D. 1, 2, 3 and 4
Explanation: The Constitution confers the following rights and privileges on the citizens of India (and
denies the same to aliens):

• Right against discrimination on grounds of religion, race, caste, sex or place of birth (Article 15)
• Right to equality of opportunity in the matter of public employment (Article 16).
• Right to freedom of speech and expression, assembly, association, movement, residence, and
profession (Article 19).
• Cultural and educational rights (Articles 29 and 30).
• Right to vote in elections to the Lok Sabha and state legislative assembly.
• Right to contest for membership of the Parliament and the state legislature.
• Eligibility to hold certain public offices, that is, President of India, Vice-President of India, judges of
the Supreme Court and the high courts, Governor of states, Attorney General of India, and Advocate
General of states.

96. With reference to the National Investigation Agency (NIA) Amendment Act, 2019, consider the following
statements:
1. The provisions of the Act extend to scheduled offenses, committed in India or abroad, that affect the
interest of India.

2. It designates state high courts as special courts for conducting the trial of offenses under the NIA Act.

Which of the statements given above is/are correct?

A. 1 only
B. 2 only
C. Both 1 and 2
D. Neither 1 nor 2
Explanation: The various features or provisions of the National Investigation Agency (NIA) Amendment Act, 2019
are as follows:

• It applied the provisions of the NIA Act also to persons who commit a scheduled offence beyond
India against Indian citizens or affect the interest of India.
• It provided that the officers of the NIA shall have similar powers, duties, privies and liabilities being
exercised by the police officers in connection with the investigation of offences, not only in India but
also outside India.
• It empowered the central government, with respect to a scheduled offence committed outside India,
to direct the NIA to register the case and take up an investigation as if such an offence has taken
place in India.
• It provided that the central government and the state governments may designate Sessions Courts
as Special Courts for conducting the trial of offences under the NIA Act.
• It inserted certain new offences in the Schedule of the NIA Act.
The Act extends to the whole of India and it applies also

• to citizens of India outside India;


• to persons in the service of the Government wherever they may be;
• to persons on ships and aircraft registered in India wherever they may be; and
• to persons who commit a Scheduled Offence beyond India against an Indian citizen or affect the
interest of India.

97. Which of the following laws are enacted for implementing the Directive Principles?
1. Payment of Bonus Act (1965)

2. Contract Labour Regulation and Abolition Act (1970)

3. Bonded Labour System Abolition Act (1976)

4. Equal Remuneration Act (1976)

Select the correct answer using the code given below.

A. 1 and 2 only
B. 2 and 3 only
C. 1, 3 and 4 only
D. 1, 2, 3 and 4
Explanation: Since 1950, the successive governments at the Centre and in the states have made several
laws for implementing the Directive Principles such as:

• the Payment of Bonus Act (1965),


• the Contract Labour Regulation and Abolition Act (1970),
• the Child Labour Prohibition and Regulation Act (1986),
• the Bonded Labour System Abolition Act (1976), to protect the interests of the labour sections.
• In 2006, the government banned child labour. In 2016, the Child Labour Prohibition and Regulation
Act (1986) was renamed the Child and Adolescent Labour Prohibition and Regulation Act, 1986.
• The Maternity Benefit Act (1961) and the Equal Remuneration Act (1976) have been made to protect
the interests of women workers.

98. Consider the following statements regarding the District Disaster Management Authority (DDMA):

1. The District Magistrate is the ex-officio chairperson of the DDMA.

2. The chief executive officer of the DDMA is appointed by the state government.

Which of the statements given above is/are correct?

A. 1 only
B. 2 only
C. Both 1 and 2
D. Neither 1 nor 2
Explanation: Every state government should establish a District Disaster Management Authority (DDMA)
for every district in the state. A DDMA consists of a chairperson and other members, not exceeding
seven. The Collector (or District Magistrate or Deputy Commissioner) of the district is the ex-officio
chairperson of the DDMA.
The chief executive officer of the DDMA, the superintendent of police, and the chief medical officer of
the district are the ex-officio members of the DDMA. The chief executive officer of the DDMA is
appointed by the state government.

99. With reference to the Goods and Services Tax Council, consider the following statements:
1. The Goods and Services Tax Council is a constitutional body.

2. Union Revenue Secretary acts as the exofficio Secretary to the Council.

Which of the statements given above is/are correct?

A. 1 only
B. 2 only
C. Both 1 and 2
D. Neither 1 nor 2
Explanation: The Constitution (One Hundred And First Amendment) Act, 2016” passed on the 8th of
September, 2016. Since then the GST council and been notified bringing into existence the Constitutional
body to decide issues relating to GST, inserted a new Article 279-A in the Constitution. This Article
empowered the President to constitute a GST Council by order. Accordingly, the President issued the
order in 2016 and constituted the Council.

As per Article 279A of the amended Constitution, the GST Council which will be a joint forum of the
Centre and the States shall consist of the following members: -

• Union Finance Minister - Chairperson


• The Union Minister of State, in charge of Revenue of finance - Member
• The Minister In-charge of finance or taxation or any other Minister nominated by each State
Government - Members
As per Article 279A (4), the Council will make recommendations to the Union and the States on important
issues related to GST, like the goods and services that may be subjected or exempted from GST, model
GST Laws, principles that govern Place of Supply, threshold limits, GST rates including the floor rates with
bands, special rates for raising additional resources during natural calamities/disasters, special
provisions for certain States, etc.

The Secretariat of the Council is located in New Delhi. The Union Revenue Secretary acts as the ex-officio
Secretary to the Council.
100. Consider the following statements with reference to the National Commission for Protection Of
Child Rights (NCPCR):
1. It is a statutory body under the administrative control of the Ministry of Women & Child Development.

2. It inquires into violation of child rights and takes suo-motu notice of matters relating to deprivation
and violation of child rights.

Which of the statements given above is/are correct?

A. 1 only
B. 2 only
C. Both 1 and 2
D. Neither 1 nor 2
Explanation: The National Commission for Protection of Child Rights (NCPCR) has been constituted by
the Government of India, under the Commission for Protection of Child Rights (CPCR) Act,2005 to
exercise and performs the powers and functions assigned to it under CPCR Act,2005. The Commission
works under the aegis of the Ministry of Women and Child Development.

The commission consists of the following members namely: A chairperson who, is a person of eminence
and has done outstanding work for promoting the welfare of children; and Six members, out of which at
least two are women are appointed by the Central Government.

The Functions of the National Commission for Protection of Child Rights as laid out in the Commissions
for Protection of Child Rights (CPCR) Act,2005 are as follows:

• Examine and review the safeguards provided by or under any law for the time being in force for the
protection of child rights and recommend measures for their effective implementation;
• Inquire into complaints and take suo-motu notice of matters relating to:
✓ Deprivation and violation of child rights;
✓ Non-implementation of laws providing for the protection and development of children;
✓ Non-compliance of policy decisions, guidelines or instructions aimed at mitigating hardships to and
ensuring the welfare of the children and provide relief to such children;
• Inspect or cause to be inspected any juveniles custodial home, or any other place of residence or
institution meant for children, under the control of the Central Government or any State Government or
any other authority, including any institution run by a social organization.
• Spread child rights literacy among various section of society and promote awareness of the safeguards
available for protection of these rights through publications, the media, seminar and other available
means;

You might also like